LSAT and Law School Admissions Forum

Get expert LSAT preparation and law school admissions advice from PowerScore Test Preparation.

 Administrator
PowerScore Staff
  • PowerScore Staff
  • Posts: 8915
  • Joined: Feb 02, 2011
|
#36352
Complete Question Explanation

Must Be True. The correct answer choice is (B)

This aerobics instructor provides that kickboxing aerobics is riskier than many other types of
exercise. This is because those without much experience in this type of aerobics often try to keep up
with the experts’ high kicks, causing overextension that can often cause injury to the hips, knees, or
lower back.

The aerobics instructor’s comments are followed by a Must be True question, so the correct answer
choice must pass the Fact Test, and be confirmed by the information provided in the stimulus. The
argument here is very straightforward, but it can help to take stock of the information you’ve been
given before considering the answer choices:
  • Premise: Beginners in kickboxing aerobics often try to keep up with the high kicks
    done by the experts, causing overextension and potential injury.

    Conclusion: Kickboxing aerobics is riskier than many other types of exercise.
Answer choice (A): The instructor does not mention whether or not the more skilled kickboxing
aerobics instructors are susceptible to injuries from overextension, so there is no way to confirm or
deny the truth of this choice based on the information provided in the instructor’s comments.

Answer choice (B): This is the correct answer choice. The injuries discussed are caused by the
overextension that comes from beginners’ trying to match the experts’ high kicks, so avoidance of
such high kicks would help to reduce the risk of injury.

Answer choice (C): While it is advisable that beginners avoid attempting the high kicks of more
skilled practitioners, such avoidance does not guarantee a beginner immunity to injury.

Answer choice (D): The information in the stimulus does not include a comparison between the risks
associated with kickboxing aerobics and the risks associated with other kinds of aerobics, so this
choice does not pass the Fact Test and cannot be the correct answer to this Must be True question.

Answer choice (E): The instructor suggests that beginners avoid attempting such high kicks, but does
not claim that the majority of beginners suffer such injury,
User avatar
 cornflakes
  • Posts: 48
  • Joined: Feb 19, 2021
|
#85460
Hi Powerscore,

I was between B and E, and properly chose B. Once I detected that we didn't know for certainty whether the majority of beginners attempt high kicks like practitioners, the answer became invalid.

Would E be correct, however, if we were to reword it in such a way that stated "most beginners at kickboxing aerobics who attempt high kicks experience injuries from them"?

Would last the sentence of the stimulus stating that overextension (and injuries stemming from) is very likely to occur for this modified subset prove with certainty that most who attempt would get injuries? In essence, does "very likely" imply more than half when applied to numerical subset?

Thanks.
 Adam Tyson
PowerScore Staff
  • PowerScore Staff
  • Posts: 5153
  • Joined: Apr 14, 2011
|
#85556
Great question, cornflakes! "Very likely" does not, by itself, imply a majority, although it can sure feel that way. Maybe a 45% chance of injury is enough to say "very likely"? Maybe even less?

And even if it was "most," all we could then conclude would be that most of those beginners would experience overextension, which we only know "often" leads to injuries. Even that doesn't necessarily mean most of the time, because "often" is a subjective term. So you could have 60% of students overextending, and even if 60% of those folks got injured, that would still be only 36% (60% of 60%) of all the ones that tried those high kicks!

Fun with numbers - the LSAT is filled with stuff like this. And I was told there wouldn't be any math!
User avatar
 lizzie18
  • Posts: 1
  • Joined: May 18, 2023
|
#101869
Hi PowerScore,

I chose A because it seemed like the best possible answer, even though I knew it didn't seem right. My reservation with B (the correct answer) was that it used "should" in the answer choice. I am weary of choosing options with "should" unless it's implied in the stimulus. Normally this leads me to the correct answer. However, on this one I was wrong to make this assumption. Can you help me with determining how"should" is allowed in this context, but not in others?

Thanks!
User avatar
 Jeff Wren
PowerScore Staff
  • PowerScore Staff
  • Posts: 385
  • Joined: Oct 19, 2022
|
#101892
Hi Lizzie,

While you definitely want to pay attention to the exact wording used in both the stimulus and the answer choices, and the verb "should" is certainly worth taking note of, it would be a mistake to rule out an answer only because of this word without analyzing the context.

The first thing to note is that the stimulus involves causal reasoning. The words "leads to" are a clue and appear on our list of causal indicator words. The aerobics instructor is essentially saying that overextending kicks often causes (leads to) injuries and then adds that such overextending often occurs when beginners try to match the high kicks of more skilled practitioners.

Based on that information, it is completely within the scope of the stimulus to conclude that beginners who wish to reduce the risk of injuries should avoid trying to match the high kicks of more skilled practitioners.

Even though the word "should" seems to be giving advice here, the advice is based on the causal reasoning in the stimulus. In other words, Answer B is basically stating "if someone wants to reduce the chance of an effect (injuries), then that person should avoid the cause (overextended high kicks). Importantly, this answer only states that avoiding the high leg kicks should reduce the chance of injuries, not that it will definitely prevent any injuries, which would be too strong/exaggerated.

A similar situation would be if the stimulus said that smoking cigarettes increases one's chances of lung cancer and the correct answer stated, "to reduce one's risk of lung cancer, one should avoid smoking cigarettes."

Get the most out of your LSAT Prep Plus subscription.

Analyze and track your performance with our Testing and Analytics Package.